necklace bead colors

This topic has expert replies
Master | Next Rank: 500 Posts
Posts: 114
Joined: Tue Mar 24, 2009 9:19 am
Thanked: 1 times

necklace bead colors

by kobel51 » Wed Jan 29, 2014 8:01 am
A necklace is made by stringing N individual beads together in the repeating pattern red bead, green bead, white bead, blue bead, and yellow bead. If the necklace design begins with a red bead and ends with a white bead, then N could equal

A) 16
B) 32
C) 41
D) 54
E) 68

User avatar
GMAT Instructor
Posts: 1052
Joined: Fri May 21, 2010 1:30 am
Thanked: 335 times
Followed by:98 members

by Patrick_GMATFix » Wed Jan 29, 2014 8:14 am
the pattern repeats itself after every five terms (red, green white, blue, yellow, red...) so if we start with a red, the last yellow bead will be the 5th, 10th, 15th, 20th... bead (a multiple of five) and the white bead that follows must be a multiple of 5 + 3 (8th, 13th, 18th, 23nd...). Only answer E makes sense. The full solution below is taken from the GMATFix App.

Image

-Patrick
Last edited by Patrick_GMATFix on Wed Jan 29, 2014 5:38 pm, edited 1 time in total.
  • Ask me about tutoring.

GMAT/MBA Expert

User avatar
Elite Legendary Member
Posts: 10392
Joined: Sun Jun 23, 2013 6:38 pm
Location: Palo Alto, CA
Thanked: 2867 times
Followed by:511 members
GMAT Score:800

by [email protected] » Wed Jan 29, 2014 5:29 pm
Hi kobel51,

These types of "repeating sequence" questions tend to be easy to beat IF you draw a picture. We're given the repeating sequence of beads: RGWBY...RGWBY...

We're asked if a string starts with R and ends with W, then the total number of beads COULD be. The phrase "could be" is important because it means that there's more than one correct answer. We have to work until we find the correct answer that's listed OR we deduce the pattern.

Following the given rules, at the minimum, we'd have...

RGW = 3 beads

That answer's not there, so keep going with the pattern:

RGWBY
RGW = 8 beads

That answer's not there either, so keep going:

RGWBY
RGWBY
RGW = 13 beads

Now, that answer isn't there either, but we now KNOW the pattern. The total increases by 5 each time, and the correct answer will end in either a 3 or an 8.

Final Answer: E

GMAT assassins aren't born, they're made,
Rich
Contact Rich at [email protected]
Image

Junior | Next Rank: 30 Posts
Posts: 15
Joined: Wed Mar 20, 2013 2:22 am
Thanked: 2 times

by rairavig » Wed Jan 29, 2014 8:15 pm
since it ends up at white, which is 3rd bead. we can make the equation as:-
3+5n= Ans
any option with unit didgit as 3 or 8 will b the Ans
i belive the right Ans is (E)